volume 24 • number 2 in this issue

24
Volume 24 • Number 2 October 2009 In This Issue Lesson 3 Toxins as Weapons Page 2 The LLSA Literature Review Page 12 Lesson 4 Rabies Page 13 The Drug Box Page 19 The Critical Image Page 21 CME Questions Page 22 The Critical ECG Page 24 Contributors Ira B. Wood, MD, Michelle M. McLean, MD, and Robert Satonik, MD, FACEP, wrote “Toxins as Weapons” Dr. Wood is a clinical instructor and senior resident at Synergy Medical Education Alliance, Michigan State University College of Human Medicine, Department of Emergency Medicine, in Saginaw, Michigan. Dr. McLean is medical director of LifeNet Michigan and on the clinical faculty at Synergy Medical Education Alliance, Michigan State University College of Human Medicine, Department of Emergency Medicine. Dr. Satonik is associate residency program director at Synergy Medical Education Alliance, Michigan State University College of Human Medicine, Department of Emergency Medicine. J. Stephen Bohan, MS, MD, FACEP, reviewed “Toxins as Weapons.” Dr. Bohan is clinical director of the Department of Emergency Medicine at Brigham and Women’s Hospital and an instructor in Medicine (Emergency Medicine) at Harvard Medical School in Boston, Massachusetts. Caroline E. Eady, MD, and Edward J. Otten, MD, wrote “Rabies.” Dr. Eady is chief resident at the University of Cincinnati Department of Emergency Medicine. Dr. Otten is a professor of emergency medicine and pediatrics and director of the Division of Toxicology at the University of Cincinnati College of Medicine in Cincinnati, Ohio. Daniel A. Handel, MD, MPH, reviewed “Rabies.” Dr. Handel is director of clinical operations in the Department of Emergency Medicine at Oregon Health & Science University, Portland, Oregon. Frank LoVecchio, DO, MPH, FACEP, reviewed the questions for these lessons. Dr. LoVecchio is research director at the Maricopa Medical Center Emergency Medicine Program and medical director of the Banner Poison Control Center, Phoenix, Arizona, and a professor at Midwestern University/Arizona College of Osteopathic Medicine in Glendale, Arizona. Louis G. Graff IV, MD, FACEP, is Editor-in-Chief of Critical Decisions. Dr. Graff is professor of traumatology and emergency medicine at the University of Connecticut School of Medicine in Farmington, Connecticut. Contributor Disclosures In accordance with ACCME Standards and ACEP policy, contributors to Critical Decisions in Emergency Medicine must disclose the existence of significant financial interests in or relationships with manufacturers of commercial products that might have a direct interest in the subject matter. Authors and editors of these Critical Decisions lessons reported no such interests or relationships. Method of Participation This educational activity consists of two lessons with a posttest and should take approximately 5 hours to complete. To complete this educational activity as designed, the participant should, in order, review the learning objectives, read the lessons, and complete the online posttest. Release date May 1, 2009. Expiration date April 30, 2012. Accreditation Statement The American College of Emergency Physicians (ACEP) is accredited by the Accreditation Council for Continuing Medical Education (ACCME) to provide continuing medical education for physicians. ACEP designates this educational activity for a maximum of 5 AMA PRA Category 1 Credits™. Physicians should only claim credit commensurate with the extent of their participation in the activity. Approved by ACEP for 5 Category I credits. Approved by the American Osteopathic Association for 5 hours of AOA Category 2-B credit (requires passing grade of 70% or better). Target Audience This educational activity has been developed for emergency physicians.

Upload: others

Post on 01-Feb-2022

2 views

Category:

Documents


0 download

TRANSCRIPT

Page 1: Volume 24 • Number 2 In This Issue

Volume 24 • Number 2

October

2009

In This IssueLesson 3 Toxins as Weapons . . . . . . . . . . . . . . . . . . . . . . . . . . . . . . . . . . . . Page 2 The LLSA Literature Review . . . . . . . . . . . . . . . . . . . . . . . . . . . .Page 12 Lesson 4 Rabies . . . . . . . . . . . . . . . . . . . . . . . . . . . . . . . . . . . . . . . . . . . . . .Page 13 The Drug Box . . . . . . . . . . . . . . . . . . . . . . . . . . . . . . . . . . . . . . . . Page 19 The Critical Image . . . . . . . . . . . . . . . . . . . . . . . . . . . . . . . . . . . . Page 21 CME Questions . . . . . . . . . . . . . . . . . . . . . . . . . . . . . . . . . . . . . . Page 22 The Critical ECG . . . . . . . . . . . . . . . . . . . . . . . . . . . . . . . . . . . . . . Page 24

ContributorsIra B. Wood, MD, Michelle M. McLean, MD, and Robert Satonik, MD, FACEP, wrote “Toxins as

Weapons .” Dr. Wood is a clinical instructor and senior resident at Synergy Medical Education Alliance, Michigan State University College of Human Medicine, Department of Emergency Medicine, in Saginaw, Michigan. Dr. McLean is medical director of LifeNet Michigan and on the clinical faculty at Synergy Medical Education Alliance, Michigan State University College of Human Medicine, Department of Emergency Medicine. Dr. Satonik is associate residency program director at Synergy Medical Education Alliance, Michigan State University College of Human Medicine, Department of Emergency Medicine.

J. Stephen Bohan, MS, MD, FACEP, reviewed “Toxins as Weapons.” Dr. Bohan is clinical director of the Department of Emergency Medicine at Brigham and Women’s Hospital and an instructor in Medicine (Emergency Medicine) at Harvard Medical School in Boston, Massachusetts.

Caroline E. Eady, MD, and Edward J. Otten, MD, wrote “Rabies.” Dr. Eady is chief resident at the University of Cincinnati Department of Emergency Medicine. Dr. Otten is a professor of emergency medicine and pediatrics and director of the Division of Toxicology at the University of Cincinnati College of Medicine in Cincinnati, Ohio.

Daniel A. Handel, MD, MPH, reviewed “Rabies.” Dr. Handel is director of clinical operations in the Department of Emergency Medicine at Oregon Health & Science University, Portland, Oregon.

Frank LoVecchio, DO, MPH, FACEP, reviewed the questions for these lessons. Dr. LoVecchio is research director at the Maricopa Medical Center Emergency Medicine Program and medical director of the Banner Poison Control Center, Phoenix, Arizona, and a professor at Midwestern University/Arizona College of Osteopathic Medicine in Glendale, Arizona.

Louis G. Graff IV, MD, FACEP, is Editor-in-Chief of Critical Decisions. Dr. Graff is professor of traumatology and emergency medicine at the University of Connecticut School of Medicine in Farmington, Connecticut.

Contributor DisclosuresIn accordance with ACCME Standards and ACEP policy, contributors to Critical Decisions in Emergency Medicine must

disclose the existence of significant financial interests in or relationships with manufacturers of commercial products that might have a direct interest in the subject matter. Authors and editors of these Critical Decisions lessons reported no such interests or relationships.Method of Participation

This educational activity consists of two lessons with a posttest and should take approximately 5 hours to complete. To complete this educational activity as designed, the participant should, in order, review the learning objectives, read the lessons, and complete the online posttest. Release date May 1, 2009. Expiration date April 30, 2012.Accreditation Statement The American College of Emergency Physicians (ACEP) is accredited by the Accreditation Council for Continuing Medical Education (ACCME) to provide continuing medical education for physicians. ACEP designates this educational activity for a maximum of 5 AMA PRA Category 1 Credits™. Physicians should only claim credit commensurate with the extent of their participation in the activity. Approved by ACEP for 5 Category I credits. Approved by the American Osteopathic Association for 5 hours of AOA Category 2-B credit (requires passing grade of 70% or better).Target Audience This educational activity has been developed for emergency physicians.

Page 2: Volume 24 • Number 2 In This Issue

Critical Decisions in Emergency Medicine

2

Lesson 3

Toxins as Weapons

Ira B. Wood, MD, Michelle M. McLean, MD, and Robert Satonik, MD, FACEP

n ObjectivesOn completion of this lesson, you should be able to:

1. Explain the importance of hospital-based decontamination in both large and small toxic exposures.

2. Describe the components of the standard cyanide antidote kit and their roles in the treatment of a patient with cyanide toxicity.

3. Discuss the presentation of a patient suffering from a vesicant exposure.

4. Explain the utility of an unremarkable chest radiograph in a patient exposed to phosgene.

5. Prescribe the appropriate antidotal and supportive therapies based on the suspected toxin.

6. Recommend the appropriate treatment plan based on elements of the history of the exposure.

7. Discuss the similarities and differences between toxic weapons and more commonly encountered toxins.

n From the EM Model17.0 Toxicologic Disorders

17.1 Drugs and Chemical Classes

Fortunately, chemical warfare is not common. However, the resulting lack of familiarity with these agents and their appropriate treatments makes their rapid identification and mitigation difficult should the need arise. When time to treatment has a significant effect on morbidity and mortality, proven generalizations and pattern recognition become tools of extreme importance.

An emerging term, weaponizable toxins, represents a paradigm shift in the way emergency physicians approach the study of and preparation for toxin-mediated multiple casualty events. Traditionally, toxins have been classified as biological or chemical. Although this distinction makes sense from the standpoint of weapons research, development, and production, it is of little or no use to the front-line emergency physician, who must make rapid decisions with limited information. Toxin exposures are all addressed through the same basic principles of limiting contact time, recognizing the toxidrome, and developing a treatment plan that involves antidotes, if available, and supportive care.

The term biologic weapon encompasses both biologic toxins such as ricin and infectious agents such as anthrax. The approach to infectious agent exposures addresses issues such as patient isolation, rapid agent identification, and the development of an appropriate antibiotic treatment regimen. Through the practical approach of emergency

medicine, it is easy to see that the treatment of a patient exposed to ricin is much more similar to the approach taken for a patient exposed to sarin gas than it is to that of a patient exposed to an infective agent such as anthrax. By reviewing biologic toxins together with chemical toxins and separating them from infectious agents, we develop a paradigm that is based on the standardized approach that must be taken in the emergency management of toxin exposures.

The terrorist attacks witnessed at the turn of this century had been previously unimaginable. It is also important to realize that emergency physicians could encounter similar toxins on a much smaller scale through accidental poisonings, law enforcement activities, or industrial accidents. The physician’s general approach and treatment priorities are the same for a patient exposed to pepper spray administered by law enforcement officials as they are for a patient exposed to a nerve agent released by terrorists.

Case Presentations

n Case OneA male college student was at a

university football game when a riot broke out. He believes he was sprayed with pepper spray. The entire event occurred about 1 hour ago. He comes through triage with complaints of burning to his skin and his eyes. Multiple staff members are also beginning to complain of burning in

Page 3: Volume 24 • Number 2 In This Issue

October 2009 • Volume 24 • Number 2

3

When is decontamination appropriate for a patient • exposed to pepper spray?Which treatment should be initiated prior to establishing • intravenous access in a patient who is unresponsive following a chemical exposure?What special concerns about methemoglobinemia in • children should influence treatment in a potential cyanide exposure?

What is the utility of a normal chest radiograph in a • patient who presents with phosgene exposure? What is the acute treatment of seizures associated • with nerve agents? Is it different from the treatment of epileptic seizures?What elements of the history and physical examination • can help determine the likely class of toxin responsible in a patient who collapses and becomes unresponsive after a chemical exposure?

Critical Decisions

their eyes. He denies any difficulty in breathing.

n Case TwoA patient is brought in from a

structure fire. The patient has suffered no burns. He was talking at the scene, complaining of headache, weakness, and dizziness, when he collapsed. EMS noted that the patient was not cyanotic but was significantly hypoxic, with a pulse oximetry of 56%. The paramedics have intubated the patient and are ventilating him with a bag valve mask. They have not established intravenous access.

n Case ThreeIt is a slow Saturday afternoon in

the department when the emergency physician receives notification from dispatch that there has been an explosion at a political rally at the downtown convention center. Almost immediately, the physician is called into the resuscitation bay to treat a patient from this explosion who was brought in by bystanders. The patient is a young man lying unresponsive on the stretcher. There are absolutely no indications of trauma. The patient seems to have copious airway secretions; his pupils are constricted, and his breathing is labored.

Chemical Agents

Incapacitating AgentsIncapacitating agents such as

pepper spray represent some of the weaponized toxins that emergency physicians are likely to encounter. Various formulations of these agents are available for both law enforcement use and personal protection. In most

cases, these agents are safe, and their adverse effects are short-lived. In rare instances there have been reports of long-term sequelae and fatalities associated with the use of these devices.1 The most commonly used agents are CS (named after its discoverers Ben Carson and Roger Staughton) and oleoresin capsicum (OC spray). They are generally dispersed as liquids or fine powders. They have also been used in hand grenades and cluster bombs.1

Symptoms from incapacitating agents begin within seconds. The cutaneous effects of CS are generally limited to a burning irritation, but there are reports of patients developing erythema and blistering.1 The patient will also experience intense lacrimation and an involuntary shutting of the eyes, known as blepharospasm. Respiratory symptoms include rhinorrhea, nasal irritation, shortness of breath, and coughing. OC, or pepper spray, is derived from plants of the genus Capsicum. The key ingredient in OC is capsaicin. Capsaicin causes the release of the neurotransmitter substance P from the peripheral afferent sensory fibers. This causes a neurogenic inflammation that is characterized by vasodilation and a sensation of pain. The effects of OC are similar to those of CS. OC tends to have a quicker onset, involves some loss of motor control, and can cause a temporary blindness. Laryngospasm and respiratory arrest have been reported.1

The management of exposures to these incapacitating toxins is not

agent specific. Decontamination is the important immediate step for all of them. Patients should be moved to a well-ventilated area, ideally with forced air blowing over them. The patient’s clothing should be removed, and the skin washed with soap and copious amounts of water. CS has a half-life of 15 minutes in water at room temperature.2 If there are ocular symptoms, the patient’s eyes may be flushed using standard techniques. Some experts favor the use of evaporative techniques such as fans or a cold hairdryer over ocular flushing.1 Contact lenses should be removed as soon as possible, and soft contacts must be discarded. Significant cutaneous erythema or dermatitis can be treated with topical steroids. Persistent respiratory symptoms could benefit from inhaled 2-agonists.

Pepper spray exposures occur frequently enough that they are often not treated with the aggressiveness that is seen in the response to other toxic exposures. Although there is reasonable variability in the potency of different preparations, all of these incapacitating agents have relatively short half-lives.

CRITICAL DECISIONWhen is decontamination appropriate for a patient exposed to pepper spray?

Although full decontamination might not be appropriate in every scenario, it should be a consideration for every patient exposed to CS or OC. If a patient is still experiencing significant symptoms of exposure after 30 minutes, continued

Page 4: Volume 24 • Number 2 In This Issue

Critical Decisions in Emergency Medicine

4

exposure should be considered, and the patient could require a more formal decontamination. If patients, visitors, and health care workers near the patient begin complaining of symptoms consistent with toxin exposure, decontamination should be performed or repeated.

Asphyxiating AgentsCyanide. Cyanide is the most

deadly weaponizable toxin that the average emergency physician is likely to encounter. It is used in many industries,3 and it is also a product of the combustion of many modern materials. It has been reported that as many as 35% of fire victims have suffered from toxic exposures to cyanide.3 The intentional use of cyanide as a poison or weapon is rare by comparison.

Cyanide works as a cellular asphyxiant by inhibiting cytochrome oxidase and preventing aerobic metabolism within the mitochondria. This forces an unsustainable shift to anaerobic metabolism, which is responsible for the classic presentation of an acyanotic patient who appears hypoxic.4 Significant exposures often lead to death well before medical care can be delivered. Patients who are still alive after a

significant exposure have a brief window of opportunity during which medical personnel can provide effective treatment.

The key to diagnosing cyanide toxicity often is the history and setting of events. Cyanide should be a consideration in any healthy patient who suddenly collapses in an environment where cyanide exposure is possible. Due to its volatility and rapid dispersion, cyanide gas is generally only effective if it is released in a closed, poorly ventilated environment. It is less likely to be the cause of a sudden collapse in an outdoor setting.

Mild cyanide toxicity will present in a manner similar to anxiety attacks. Patients may be hyperventilating and complaining of shortness of breath. Other symptoms include headaches, weakness, and dizziness. Because of their dependence on oxygen, the central nervous system and the cardiovascular system are generally the first to manifest symptoms of an exposure.

The standard cyanide antidote kit in the United States contains amyl nitrite, sodium nitrite, and sodium thiosulfate (Table 1). The nitrites are used to convert hemoglobin

to methemoglobin, which has a stronger affinity for cyanide and facilitates the removal of cyanide from the mitochondria. The sodium thiosulfate acts as a sulfhydryl donor for rhodanese. Rhodanese is the enzyme responsible for converting cyanide to thiocyanate, a much less toxic metabolite that is then renally excreted. Thus the treatments are complementary.

CRITICAL DECISIONWhich treatment should be initiated prior to establishing intravenous access in a patient who is unresponsive following a chemical exposure?

Amyl nitrite from a standard antidote kit should be administered. It is packaged in glass pearls. In patients requiring assisted ventilations, the amyl nitrite can be delivered via bag valve mask by placing a crushed pearl over the oxygen intake valve. When this method is used, the amyl nitrite should be used for only 30 seconds of every minute. Replace the pearl every 3 minutes.5 Alternatively, the pearls may be crushed into a gauze sponge and held under the patient’s nose.6 Theoretically, this can induce a methemoglobinemia of 3% to 5%,5 but this may not be reliable in clinical

Therapy Adult Dosing Pediatric Dosing Notes

Amyl nitritea Crushed into gauze and inhaled by patient

Crushed into gauze and inhaled by patient

Induces 3% to 5% methemoglobinemia, but not reliable . Provides potential for rapid therapy prior to obtaining intravenous access

Sodium nitritea 300 mg IV push over 2 to 4 minutes

10 mg/kg, or 6 mg/kg Consider sequelae of methemoglobinemia in anemic children

Sodium thiosulfatea 50 mL of 25% solution 1 .65 mL/kg of 25% solution over 10 minutes

Hydroxocobalamin 5 gm over 15 minutes 70 mg/kg Now available as a cyanide antidote in the United States . Other forms are too dilute for this therapy

Table 1.Summary of cyanide antidotal therapies

aComponents of the traditional cyanide antidote kit used in the United States

Page 5: Volume 24 • Number 2 In This Issue

October 2009 • Volume 24 • Number 2

5

practice. Amyl nitrite is a bridging therapy that can be instituted rapidly. Once venous access has been established, other, more reliable therapies should be instituted.

Sodium nitrite is more effective than amyl nitrite and may raise methemoglobin levels to over 20%.7

Sodium nitrite is given as 300 mg (10 mL of a 3% solution) intravenously over 2 to 4 minutes.

CRITICAL DECISIONWhat special concerns about methemoglobinemia in children should influence treatment in a potential cyanide exposure?

In children, there is significant concern about the potential decreased oxygen-carrying capacity associated with methemoglobin. Assuming a child has a normal hemoglobin level, the recommended dose of sodium nitrite is 10 mg/kg. Since time is of the essence, it may be unreasonable to wait for a hemoglobin level before initiating treatment in a child presenting with cyanide toxicity. In those instances, it may be prudent to administer sodium nitrite in a dose of 6 mg/kg, as this dose would be safe, even in a child with a significant anemia.5

These traditional nitrite therapies have several drawbacks. If given too aggressively, they can cause significant hypotension and tachycardia. Additionally, the benefits of methemoglobin come at the price of an overall decreased oxygen-carrying capacity. This is especially concerning in patients with additional impairments of tissue oxygenation such as carbon monoxide poisoning. For this reason, some experts recommend avoiding nitrite therapy in victims of smoke inhalation, although this is debateable.5

Sodium thiosulfate works synergistically with the nitrites to rid the body of its toxic cyanide load. Rapid administration can cause hypotension, but otherwise this is a fairly safe therapy, with rare reports of hypersensitivity. Sodium thiosulfate

works with the patient’s intrinsic enzymes to detoxify cyanide. The metabolite is renally excreted. This can cause toxicity in patients with significant renal failure. In severe circumstances, the medication could be given and then dialyzed.5 The adult dose of sodium thiosulfate is 12.5 grams (50 mL of a 25% solution). In children, the dosage is 1.65 mL/kg of the 25% solution, given over 10 minutes.

Hydroxocobalamin is a therapy that has been used for decades in Europe and is now available in the US market (Table 1). Hydroxocobalamin combines with cyanide to form cyanocobalamin, or vitamin B12. This therapy is generally used in place of the nitrite therapy, and is also synergistic with sodium thiosulfate. Hydroxocobalamin does not hinder the blood’s oxygen-carrying capacity, making it an especially attractive option for smoke inhalation victims. The adult dosage is 5 grams over 15 minutes. This dose will bind up to 250 grams of cyanide. The pediatric dose is 70 mg/kg. Hydroxocobalamin is also used for the treatment of pernicious anemia. It is important to realize that the drug preparation used for pernicious anemia is diluted to such an extent that it has no utility as an antidote for cyanide toxicity. It is also important to note that sodium thiosulfate will bind to hydroxocobalamin, so the two medications should be given through different veins, if possible.5

During an exposure event, it is likely that the emergency physician’s choice of antidote will be dictated by what is readily available. The best antidote is generally the one that is on hand. Obviously, supportive care that focuses on enhancing tissue oxygenation through blood pressure maintenance and supplemental oxygen therapy is of paramount importance.

Phosgene. Phosgene is representative of the asphyxiating agents that work through direct effects on the pulmonary tissues, as

opposed to the cellular asphyxiants like cyanide. Phosgene, or carbonic dichloride, is a synthetic chemical commonly used in the pharmaceutical, pesticide, plastics, and dye industries. Phosgene can also be created through the heating of Freon gas. It is heavier than air and said to have an odor similar to that of freshly cut corn or hay. Phosgene was considered the most lethal chemical warfare agent in World War I.4

The classic presentation of phosgene exposure involves some initial mild irritation of the nasal and pulmonary passages. This is then followed by a symptom-free period of 2 to 10 hours before the onset of more significant pulmonary symptoms.6

CRITICAL DECISIONWhat is the utility of a normal chest radiograph in a patient who presents with phosgene exposure?

It is common for chest radiographs to appear unremarkable for the first several hours following an exposure to phosgene. In fact, dyspnea, or radiographic evidence of pulmonary edema within 4 hours of exposure is associated with a significant exposure and probably warrants admitting the patient to an ICU setting.8 Conversely, patients are unlikely to develop significant lung injury if they are asymptomatic and have a clear chest radiograph 8 hours after exposure.8 An unpredictable minority of patients can develop significant symptoms after 48 hours, leading to the recommendation that exposed individuals be observed for 48 hours postexposure.7

Care of the patient exposed to phosgene or the other primary lung toxins is primarily supportive. There are no specific antidotes. Bed rest is important because postexposure exertion appears to be related to a worsening of the inflammatory response.8 Bronchodilators are effective, but corticosteroid therapy remains controversial. Diuretics should be avoided because they will exacerbate the hypovolemia

Page 6: Volume 24 • Number 2 In This Issue

Critical Decisions in Emergency Medicine

6

associated with this noncardiogenic pulmonary edema.6 Superimposed infection can be a major contributor to morbidity and mortality, but prophylactic antibiotic therapy is not recommended.8 The mainstay of therapy is volume replacement and ventilatory support.6

Nerve AgentsNerve agents were fist synthesized

as insecticides by German scientists in 1937. They were stockpiled as potential weapons by Germany during World War II but were not used in war until the Iran-Iraq conflict of the 1980s.9 These agents are relatively simple and inexpensive to synthesize, making them potentially attractive weapons for terrorists.7 There are five primary nerve agents that are used as chemical weapons. They are identified by a common name and a two-letter North Atlantic Treaty Organization (NATO) designation (Table 2). Most nerve agents are very volatile and will vaporize at room temperature. The agent VX is the exception to this; it requires a temperature above 38ºC (100.4°F) to cause a significant vapor hazard.10 All nerve agents are cholinesterase inhibitors; therefore, the clinical presentation of a nerve agent exposure can be similar to other, more common cholinergic crises. When compared with organophosphate insecticide exposures, nerve agent exposures generally have a much quicker onset and a shorter overall duration of effect and require a lower total dose of atropine.8

The diagnosis of nerve agent intoxication is based entirely on clinical presentation and exposure history. A scenario in which several people in close proximity to one another develop both respiratory and central nervous system abnormalities is highly suggestive of nerve agent exposure.10 Blood cholinesterase activity studies are not useful in the acute management of these patients.5

The route of exposure significantly influences the clinical presentation. Inhalation exposures present rapidly and with classic signs and symptoms. Dermal exposures, however, might not cause symptoms for up to 18 hours.9 Isolated dermal exposures can present with localized diaphoresis and fasciculations. Miosis can be absent or significantly delayed in patients with pure dermal exposures. Patients with pure vapor exposures who do not develop symptoms early are very unlikely to have a significant exposure.9

The treatment of a patient with a significant nerve agent exposure has three primary components: supportive care, respiratory support, and seizure control. Supportive care is based on the specifics of the exposure and the patient’s presentation along with an appreciation for the possible pathways of deterioration. Specific concerns center on respiratory support and the prevention and treatment of seizures. Atropine is given as a competitive inhibitor of the muscarinic receptors to lessen the respiratory effects of cholinergic stimulation. Oximes are chemical compounds that if given early enough are able to disrupt the chemical bond between the nerve agent and the acetylcholinesterase (AChE) active site, thus allowing reactivation of the enzyme.

Atropine is not indicated for patients presenting with miosis

as their only symptom. For adults, atropine should be given intravenously at a dose of 2 mg every 2 to 5 minutes until the patient is breathing easier, or, for intubated patients, until the patient becomes less difficult to ventilate. In the unconscious adult, a higher initial dose of 6 mg should be given. For children, the dose should be 0.05 to 0.1 mg/kg up to 4 mg given every 5 to 10 minutes. It is important to note that the more familiar symptomatic bradycardia dose of 0.02 mg/kg is not a sufficient treatment for children with nerve agent intoxication.10

Pralidoxime chloride (2-PAM) is the only oxime approved for nerve agent treatment in the United States. If given early enough, it is able to remove the nerve agent from the AChE, and form a complex with the nerve agent that is then excreted in the urine. If pralidoxime is given too rapidly, significant hypertension can occur. It should be given to adults at 1 to 2 gm IV drip over 30 minutes; it may then be given as a continuous drip at 500 mg/hour. For children, the dose is 15 to 25 mg/kg as a slow IV push or IM injection, up to 600 mg.

The effectiveness of 2-PAM is time dependent. Nerve agents undergo a process known as aging, during which they essentially develop a covalent bond with the AChE. Once this bond is formed, 2-PAM is unable to remove the nerve agent from the

Common Name NATO Designation Characteristics

Soman GD Ages in 2 to 6 minutes,2 camphor-like odor3

Sarin GB 50% aged in 5 hours,4 odorless; most volatile

Tabun GA Fruity odor3

Cyclosarin GF

VX VX 50% aged at 38 hours,5 clear amber color; oily; least volatile; most persistent

Table 2.Common nerve agents

Page 7: Volume 24 • Number 2 In This Issue

October 2009 • Volume 24 • Number 2

7

AChE. Aging occurs at different rates for different nerve agents (Table 2). Soman has the shortest aging time (a few minutes), while VX has very little aging effect.7

Aging is not a contraindication to the use of oxime therapy. If AChE is not reactivated by oxime therapy, it will need to be replaced through de novo synthesis. This process can take several weeks.5 Although aging can reduce the effectiveness of oxime therapy, any AChE regeneration will be beneficial to the patient.

Seizures can occur in the face of a significant nerve agent exposure. Seizures are thought to be related to the cholinergic hyperstimulation. Patients with significant nerve agent exposures can also present with flaccid paralysis. Patients with paralysis should have continuous EEG monitoring to detect the presence of nonconvulsive seizures.

CRITICAL DECISIONWhat is the acute treatment of seizures associated with nerve agents? Is it different from the treatment of epileptic seizures?

High-dose atropine might terminate seizure activity if given within the first 5 minutes of the seizure. After 5 minutes the seizure will begin causing biochemical changes within the brain that will make anticholinergic therapy alone ineffective, and benzodiazepines must be employed. Midazolam

appears to be slightly more effective and is the benzodiazepine of choice for these seizures, but others are acceptable. Diazepam is available in 10-mg auto-injectors. When choosing between midazolam, lorazepam, and diazepam, the drug of choice in this scenario is the drug on hand. Pentobarbital may be used if the seizure is refractory but can cause significant respiratory depression. Fosphenytoin does not have a role in the treatment of nerve agent-related seizures.5

Ocular complaints are a significant component of the presentation when patients are exposed to nerve agents. Miosis was present in 99% of the patients with moderate or worse exposures when sarin was released into the Tokyo subway system.5 Note that miosis is not as reliable an indicator of exposure in children.10 In addition to the constricted pupils, there is also ciliary muscle contraction that causes pain. Patients may get some relief from these ocular manifestations with tropicamide drops, 1 to 2 drops of a 0.5% solution.

Prepackaged auto-injectors containing atropine and 2-PAM are available for military and prehospital use. The kit used in the United States is the Mark 1 nerve agent auto-injector kit. The Mark 1 kits are also part of the Centers for Disease Control and Prevention (CDC) CHEMPACK project, which aims to rapidly deploy stockpiled resources

to the areas of need in the event of a mass casualty event.5 The kit contains a 2-mg atropine auto-injector, and a 600-mg pralidoxime auto-injector. For medication administered via intramuscular injection, the auto-injectors have been shown to be more effective than traditional intramuscular injection methods.5 In the military arena, soldiers are instructed to administer three Mark 1 kits, delivering a total of 6 mg of atropine and 1,800 mg of 2-PAM. If symptoms persist, they administer only the atropine syringe from additional kits.5 Diazepam auto-injectors are also available in 10-mg doses.

Pediatric auto-injectors are also available for the administration of atropine (Table 3). They come in 0.5-mg and 1-mg doses. Children in the 20- to 40-kg weight range should receive the 1-mg dose; children who weigh less than 20 kg should get a 0.5-mg dose, and children weighing more than 40 kg can receive the 2-mg adult dose.5 Although age- and weight-appropriate dosing is always ideal, it is not always possible in mass casualty scenarios with taxed resources. Some authors have suggested that in children with symptomatic nerve agent intoxication, the toxic effect of over-medication with atropine or 2-PAM would present fewer sequelae than failure to treat the nerve agent intoxication.10

VesicantsMustard agents are clear, odorless,

oily substances that vaporize easily. Although there are several types of “mustard agents,” sulphur mustard is the agent generally used as a weapon. This agent was used in World War I and was referred to as the “king of the battle gases.”11 More recently, sulphur mustard gas appears to have been used by Iraq, during its conflict with Iran in the late 20th century. The exact toxic mechanisms of sulphur mustard are not clear, but it appears to undergo an alkylation reaction with cellular components, including

Weight Therapy

Atropine

< 20 kg 0 .5-mg pediatric auto-injector

20 to 40 kg 1-mg pediatric auto-injector

> 40 kg 2-mg adult auto-injector

2-PAM

> 12 kg 600-mg adult auto-injector

Diazepam

> 30 kg 10-mg adult auto-injector

Table 3.Pediatric use of auto-injector therapies for nerve agent exposures

Page 8: Volume 24 • Number 2 In This Issue

Critical Decisions in Emergency Medicine

8

DNA and RNA. Research into these mechanisms has found several nitrogen mustards that are used in the treatment of cancers.12

Sulphur mustard is generally found as a transparent brown liquid, with an odor similar to that of garlic. The color and odor are the result of impurities in the manufacturing process. In vapor form, it will rapidly pass through clothing to the underlying skin. It has also been shown to penetrate wood and leather barriers.11 The vapor has much better skin penetration than the liquid. Up to 80% of liquid mustard will evaporate from the skin, leaving only 20% of the agent available for penetration.12

Sulphur mustard exposure has a delayed presentation when compared to most chemical weapons. Ocular symptoms such as lacrimation, a foreign body sensation, and conjunctivitis develop 4 to 12 hours postexposure and are some of the earliest clinical manifestations of mild exposures.12 In severe exposures edema, blepharospasm, and transient blindness can appear within 3 hours of exposure.12 There is an inverse relationship between the severity of the exposure and the delay in clinical presentation.

Skin manifestations will begin with redness and pruritus in 4 to 8 hours. The classic blisters and Nikolsky sign are generally not apparent for at least 2 days. Skin lesions can be treated like second-degree burns, with the understanding that they will often take much longer to heal than thermal burns. The fluid requirements are the same as for burn patients—with an important exception. The fluid loss in a sulphur mustard exposure does not occur until blister formation; therefore aggressive fluid therapy begins with blister formation and not over the first 24 hours as in thermal burns.12

Pulmonary manifestations following mustard gas exposure can be rapidly fatal. An early presentation clinically similar to bronchitis

should be considered a sign of direct pulmonary injury. In patients with stridor and hoarseness, early tracheostomy should be considered. Laryngospasm is a persistent threat. Sulphur mustard exposure is also associated with the formation of pseudomembranes, similar to those seen in diphtheria, which can cause fatal obstructions.12

Systemic effects can cause a profound immunosuppression with leukopenia. This usually peaks on days 7 to 9.12 Significant long-term effects tend to involve the pulmonary system. The most common long-term pulmonary manifestation is a chronic bronchitis.12

Decontamination only limits patient exposure if performed within the first few minutes. Water should be used in copious amounts. Military forces have specific decontamination formulations. In a civilian setting, there may be some benefit to the use of talcum powder or flour for decontamination.12

Treatment for sulphur mustard exposures is primarily supportive. Cutaneous lesions should be treated like second-degree burns. Respiratory support and anticipation of deterioration are important aspects of the treatment plan.

Biologic Toxins

Botulinum ToxinBotulinum toxin is one of the

most toxic substances known and is relatively simple to produce. One gram of pure toxin, if effectively dispersed as an aerosol, could kill up to 1 million people.4 Inhalation exposure does not occur in nature, and case reports of accidental industrial exposures are limited. Food contamination is also a potential source of exposure.

The classic, acute presentation of botulinum intoxication involves bilateral cranial nerve palsies and descending motor paralysis. Initially, a bulbar paralysis is seen, followed by the loss of head control and a descending skeletal muscle paralysis.

Ingestion symptoms generally develop within 12 to 72 hours, but there are reports of delays of up to 1 week.4 Although the data are limited, it is thought that inhalation exposures would follow a similar time course. There is no change in mentation, and there should not be a fever. Blood, cerebrospinal fluid, and imaging study results are generally unremarkable.4 Intoxication may be confused with Guillian-Barré syndrome, myasthenia gravis, and stroke.4

Treatment is based on supportive therapy and the administration of antitoxin. Antitoxin will not reverse preexisting paralysis, but it will halt further progression. A trivalent antitoxin, covering serotypes A, B, and E is available in the United States through the CDC and state health agencies. An experimental heptavalent antitoxin is available to the US military. The antitoxin has a half-life of several days and is administered in a single, one-vial dose. The currently approved antitoxin is equine derived, and therefore, a small test dose is advised prior to administering the full dose to allow for the identification and pretreatment of allergic individuals.

RicinRicin is a toxin derived from the

bean of the plant Ricinus communis (castor bean). It binds to the 28S ribosomal subunit, interfering with protein synthesis. Annually, there are 1 million tons of castor beans processed into castor oil worldwide.4

The waste mash from this process is 5% ricin by weight.4 The industrial extraction process leaves no active ricin in the oil.13 The beans themselves are generally only toxic if they are macerated or chewed to release the toxin, although death has been reported with as few as 2 beans.13 Unfortunately, there are no specific treatments for ricin toxicity, and supportive care is the only currently available option.13

Ricin has been used as a criminal toxin in recent history. Ricin has been

Page 9: Volume 24 • Number 2 In This Issue

October 2009 • Volume 24 • Number 2

9

found in the mailroom of a US senator and in a letter to the White House. Although it was never proved, ricin is the likely agent that killed Bulgarian defector Georgi Markov in September of 1978 when it was injected into him as a small pellet from the modified tip of a common umbrella.

Ingestion of ricin leads to necrosis of the gastrointestinal epithelium. There is associated hepatic, splenic, and renal necrosis.4 This progresses to significant fluid losses, bleeding, and shock. Death often occurs after the second day. The initial presentation is nonspecific, with colicky abdominal pain, vomiting, and diarrhea. Laboratory abnormalities can reflect renal and hepatic involvement. Symptom onset is generally within 4 to 6 hours, but can be as long as 10 hours.13

Injection exposures will present with local tissue damage as well as necrosis of local and regional lymph nodes.4 This can then progress to involvement of the gastrointestinal tract, liver, and kidneys. Disseminated intravascular coagulation and third-degree heart block could also develop.14 Onset of symptoms can take up to 12 hours after an exposure.13

There is poor information on inhalational exposure of ricin in humans. Limited case reports show fever, arthralgias, chest tightness, and respiratory symptoms.13 Eventually there can be pulmonary necrosis and the development of pulmonary edema.14 Symptom onset can be delayed up to 24 hours.

Response to Toxic DisastersAnalyses of mass casualty

incidents reveal that the same mistakes tend to be repeated time and time again.15 Hospital personnel tend to believe that their role in a mass casualty chemical exposure will be to treat multiple victims of a known exposure.15 It is also commonly believed that most of the victims will be brought to the hospital by the local EMS system and will

have been decontaminated prior to their arrival.15 History clearly shows otherwise.

The 1995 release of sarin in the Tokyo subway system provides important insight.16 At the closest hospital, only 7% of the victims arrived by ambulance; 59% of the patients either walked or arrived by taxi.16 Three victims suffered prehospital cardiac arrest. Two of the three were taken by private vehicles to hospitals that were farther away.16 There was a 20% secondary contamination rate among nurses and physicians.16 The toxin was not identified for several hours, and once it was identified, the information was not disseminated effectively to the local hospitals. Within the hospital, the correct diagnosis was made based on clinical presentation and specialist consultation via telephone.16

It is clear that this sort of incident might not be recognized before patients have arrived at an emergency department. As a result, there is a risk of contamination within the emergency department before a mass exposure event has even been recognized by the health care system. It is imperative that disaster planning include a contingency plan to deal with facility and personnel contamination.

Decontamination is of paramount importance any time a chemical exposure is expected. The decision to decontaminate should be made by frontline personnel and not postponed for physician evaluation. It is always better to err on the side of decontamination than to unnecessarily expose personnel to potentially deadly toxins. Decontamination should occur with tepid water. Warm water can cause vasodilation that could enhance the dermal absorption of toxins.7

Empiric TreatmentsEmpiric treatment in the face of

significant toxin exposures will save lives. This requires that emergency physicians make important treatment

decisions rapidly with limited information. This type of medical decision making is the foundation of emergency medicine as a specialty.

CRITICAL DECISIONWhat elements of the history and physical examination can help determine the likely class of toxin responsible in a patient who collapses and becomes unresponsive after a chemical exposure?

Nerve agents and cyanide toxicity require immediate unrelated treatments and have similar clinical presentations in significant exposures. Cyanide toxicity would be more likely in an industrial setting and generally requires an enclosed, poorly ventilated environment to induce a rapid and dramatic onset; its rapid dispersal limits its effect in well-ventilated areas. Nerve agents are more likely to present as part of a criminal release, with the intent to harm several persons. These agents can be effective even when widely dispersed over open areas. An isolated nerve agent exposure is possible, however, especially during the manufacture and preparation of the agent.

Nerve agents present with classic cholinergic symptomology. This involves miosis, diaphoresis, salivation, lacrimation, fasciculations, and bronchorrhea. In severe cases there can be a flaccid paralysis. Cyanide is a cellular asphyxiant. It causes a nonpulmonary hypoxia. Cyanide is not associated with the secretions or fasciculations seen in nerve agent exposures.8 Patients exposed to cyanide will have a high anion-gap acidosis and above normal venous oxygen saturation.

Case Resolutions

n Case OneThis patient was moved to a

well-ventilated area. His clothing was removed, and his skin was washed with soap and water. Some erythema was seen on his arms, which had been exposed.

Page 10: Volume 24 • Number 2 In This Issue

Critical Decisions in Emergency Medicine

10

Hydrocortisone cream was applied following decontamination. The patient’s contact lenses were removed and discarded. Morgan lenses were placed, and his eyes were flushed with room-temperature saline. Fluorescein stain was applied to each eye, and a slit-lamp examination revealed no corneal abrasions. He was monitored for 45 minutes, and his symptoms completely resolved. He was discharged, with hydrocortisone cream for his skin eruption.

n Case TwoBecause this patient collapsed

after being rescued from a fire that occurred in a clothing factory with wool and silk products, cyanide poisoning was suspected. A cyanide antidote kit was obtained, and an amyl nitrite glass pearl was crushed

and placed over the oxygen intake valve of the bag valve mask. It was used for 30 seconds of every minute, and a new pearl was placed every 3 minutes. Intravenous access was then obtained, and sodium nitrite was administered at 300 mg over about 3 minutes. The patient was placed on a cardiac monitor and monitored for hypotension and tachycardia. The patient had no known history of renal failure, and a 12.5-gram dose of sodium thiosulfate was administered. The patient’s laboratory values were monitored, and the methemoglobin level was 15%. He was admitted to the ICU.

n Case ThreeThis presentation and history

of events was suspicious for an intentional toxin release, specifically,

the release of a nerve agent. The emergency physician realized that the emergency department had become a “hot zone,” and that he and the staff were now potential victims. The institutional response protocol was initiated. Emergency department staff quickly assembled a rapid-response decontamination team and began decontamination of the patient and exposed staff. The hospital was notified that this exposure was an intentional release of a vaporized nerve agent. Minimal cross-contamination of staff was expected. The patient and contaminated staff were treated with atropine and 2-PAM.

The hospital initiated the mass casualty plan and took measures to increase surge capacity. Unfortunately, this plan had not addressed the possibility that the emergency department would become contaminated and be forced to close. Additional staff members were called in. Because it was a Saturday and there were no scheduled surgeries, the emergency physician was able to move patients from the observation unit to the preoperative holding area, where they could still be appropriately monitored. This enabled conversion of the observation unit into a temporary emergency department.

In the end, the toxic agent was poorly dispersed, and the hospital received only a handful of patients. However, this scenario revealed several flaws in the hospital’s response plan. The most important problem was the lack of a contingency plan that addressed contamination and closure of the emergency department.

Summary Toxic weapons exposures are

uncommon events; however, when presented with the victim of a possible toxic weapon, emergency physicians must act quickly and decisively. Decisions made within the first few minutes of patient contact can have a significant effect on the

Pearls and Pitfalls Children can be less tolerant of the methemoglobinemia associated with nitrite •therapy for cyanide toxicity than adults, especially if they are anemic. If a child’s hemoglobin level is unknown, a dose of 6 mg/kg of sodium nitrite may be given. Nitrite therapy may also be withheld. Hydroxocobalamin therapy is not associated with these concerns.

Patients who have abnormal chest radiographs or experience respiratory •distress within 4 hours of a phosgene exposure are likely to have significant deterioration and warrant admission to an ICU.

Although the aging of a nerve toxin can limit the •effectiveness of an antidote, antidote treatment is still likely to benefit the patient and should not be withheld.

Children exposed to a nerve agent are less likely •to develop miosis than are adults.

Because there are rare case reports of patients developing significant •pulmonary edema up to 48 hours after their exposure to phosgene gas, some experts recommend a 48-hour observation period for these patients.

Inhalation exposures take effect rapidly, but the effects of •dermal exposures can be delayed up to 8 hours.

Most victims of toxic disasters do not arrive by ambulance; hospital disaster •plans should address the possibility that there will be a delay in recognizing the event and that the emergency department could become contaminated.

A patient who collapses and becomes unresponsive after a chemical exposure •has likely been exposed to either a nerve agent or cyanide. Patients exposed to nerve agents present with miosis, diaphoresis, salivation, lacrimation, fasciculations, bronchorrhea, or even flaccid paralysis; patients exposed to cyanide will present with none of the classic cholinergic symptomology.

Page 11: Volume 24 • Number 2 In This Issue

October 2009 • Volume 24 • Number 2

11

outcome for the patient, as well as the community. Successful management will depend on recognition of a potential exposure and identification of the likely toxicant. Timely toxin identification is likely to rely on the clinical acumen and resource utilization of the emergency physician. Prehospital and public safety personnel may not recognize the threat before transporting patients to the emergency department; the first suspicion of a toxin exposure might not arise until after the first patients have arrived in the emergency department. It is imperative that response plans to toxic exposures do not rely entirely on advance notification from prehospital resources.

ReferencesSmith J, Greaves I. The use of chemical incapacitant 1. sprays: a review. J Trauma. 2002;52(3):595-600.

Blain PG. Tear gases and irritant incapacitants. 2. 1-chloroacetophenone, 2-chlorobenzylidene malononitrile and dibenz[b,f]-1,4-oxazepine. Toxicol Rev. 2003;22(2):103-110.

DesLauriers CA, Burda AM, Wahl M. 3. Hydroxocobalamin as a cyanide antidote. Am J Ther. 2006;13(2):161-165.

Greenfield RA, Brown BR, Hutchins JB, et 4. al. Microbiological, biological, and chemical weapons of warfare and terrorism. Am J Med Sci. 2002;323(6):326-340.

Lawrence DT, Kirk MA. Chemical terrorism attacks: 5. update on antidotes. Emerg Med Clin North Am. 2007;25(2):567-595.

Muskat PC. Mass casualty chemical exposure and 6. implications for respiratory failure. Respir Care. 2008;53(1):58-63; discussion 63-66.

Fry DE. Chemical threats. 7. Surg Clin North Am. 2006;86(3):637-647.

Kales SN, Christiani DC. Acute chemical emergencies. 8. N Engl J Med. 2004;350(8):800-808.

Lee EC. Clinical manifestations of sarin nerve gas 9. exposure. JAMA. 2003;290(5):659-662.

Baker MD. Antidotes for nerve agent poisoning: 10. should we differentiate children from adults? Curr Opin Pediatr. 2007;19(2):211-215.

Rice P. Sulphur mustard injuries of the skin. 11. Pathophysiology and management. Toxicol Rev. 2003;22(2):111-118.

Kehe K, Szinicz L. Medical aspects of sulphur mustard 12. poisoning. Toxicology. 2005;214(3):198-209. Epub 2005.

Audi J, Belson M, Patel M, et al. Ricin poisoning: a 13. comprehensive review. JAMA. 2005;294(18):2342-2351.

Madsen JM. Toxins as weapons of mass destruction. 14. A comparison and contrast with biological-warfare and chemical-warfare agents. Clin Lab Med. 2001;21(3):593-605.

Kirk MA, Deaton ML. Bringing order out of chaos: 15. effective strategies for medical response to mass chemical exposure. Emerg Med Clin North Am. 2007;25(2):527-548.

Tokuda Y, Kikuchi M, Takahashi O, Stein GH. 16. Prehospital management of sarin nerve gas terrorism in urban settings: 10 years of progress after the Tokyo subway sarin attack. Resuscitation. 2006;68(2):193-202.

Page 12: Volume 24 • Number 2 In This Issue

Critical Decisions in Emergency Medicine

12

The LLSA Literature Review“The LLSA Literature Review” summarizes articles from ABEM’s “2010 Lifelong Learning and Self-Assessment Reading List.”

Many of these articles are available online in the ACEP LLSA Resource Center (www.acep.org/llsa) and on the ABEM web site.

Disclosing Harmful Medical Errors to PatientsReviewed by Pooja Agrawal, MD, and J. Stephen Bohan, MD, MS, FACEP; Harvard Affiliated Emergency Medicine Residency; Brigham & Women’s Hospital

Gallagher TH, Studdert D, Levinson W. Disclosing harmful medical errors to patients. N Engl J Med. 2007;356:2713-2719.

Medical errors are common. Even with ongoing and ap-propriate efforts to decrease the incidence of medical errors, some amount of error is unavoidable. Patients are increas-ingly expecting to be informed if a medical error occurs. Phy-sicians should be trained in and become comfortable with discussing harmful medical errors with their patients.

There are several models for addressing disclosure of med-ical errors. The most obvious is that the responsible clinician has the necessary conversation with the patient involved. In order for this to be effective, clinicians need to get basic in-struction on how to handle these situations; simulation can be used as an educational tool. Another model involves risk managers or medical directors who would coach the physi-cian when disclosure is warranted. In another model, the involved clinician is moved to the periphery and a “rapid response team” conducts the necessary disclosure. There is, however, concern as to how this last method might affect the patient-physician relationship.

Many physicians have not been trained to have this kind of conversation with patients and are not comfortable doing so. This has been a barrier to open disclosure. Another sig-nificant barrier is the physician’s fear of litigation if an error is revealed. Will an apology or disclosure be considered a legal admission of responsibility for an error? What effect will disclosure have on patient satisfaction? These questions are as yet unanswered.

The Joint Commission issued its first nationwide disclo-sure standard in 2001 and further clarified the standard in 2006. These standards strongly recommend that patients be informed about all care outcomes, including potentially harmful or unanticipated ones. They link compliance with this standard to hospital accreditation. As a result, 69% of hospitals evaluated in 2005 had established disclosure poli-cies.

Many states do not consider disclosure of a medical er-ror as an admission of guilt. Several government and private

legal initiatives promote disclosure of medical errors and de-emphasize legal liability; however the effect of disclosure on litigation is still largely unknown. Prominent disclosure programs that have been operating for some time report a significant reduction in the cost and frequency of litigation after their programs began; however, these findings have not been rigorously assessed or validated.

The widespread implementation of disclosure policies is inevitable. There are nuances of this topic that must be better understood, including the best method for disclosing error and the associated medical and legal implications. In the fu-ture, it is likely that full and open disclosure of medical errors to patients will become standard practice.

HighlightsMedical errors are inevitable and must be addressed • appropriately.

There are several models for disclosure of medical errors; in • all cases, appropriate physician education is necessary.

The legal implications of disclosing a medical error are not • defined; however, initial data suggest that admitting to an error does not increase rates of litigation.

Page 13: Volume 24 • Number 2 In This Issue

13

Lesson 4

n ObjectivesOn completion of this lesson, you should be able to:

1. Discuss routes of rabies transmission.

2. Describe the importance of early postexposure prophylaxis.

3. Describe the process for definitively identifying rabies infection.

4. Detail appropriate treatment regimens following rabies exposure.

5. Identify symptoms of rabies infection.

n From the EM Model10.0 Systemic Infectious Disorders

10.6 Viral

Caroline E. Eady, MD, and Edward J. Otten, MD, FACMT, FAWM

Rabies

Although rabies is not a common disease in the United States, it is still among the leading infectious killers worldwide, claiming 40,000 to 70,000 lives each year.1,2 This disease affects a disproportionate number of children, who are too young to understand the dangers of wild animals and do not report exposures to their parents.3,4

Rabies is a single-stranded RNA virus of the Lyssavirus genus.1,5 It is carried by wild animals and is transmitted to humans through exposure to infected saliva, most often from a bite.1,2 It is a fatal virus, so early identification of an exposure is important to allow for neutralizing immunoglobulin and vaccine administration.6

An estimated 40,000 people will present with a request for a rabies vaccine in the United States this year.3 There is a fixed supply of vaccine available, so practitioners must be able to determine who is truly at risk so that they can ration supplies accordingly. Beginning in June 2007, the Sanofi Pasteur plant, maker of the IMOVAX rabies vaccine, has been undergoing renovations to comply with FDA requirements for such facilities. Because of this, production has been drastically reduced, and Novartis, who produces the RabAvert vaccine, has not been able to meet the current demand, which has increased from previous years. The renovations for the Sanofi Pasteur plant are scheduled to be finished by the end of 2009.7

Emergency physicians must also be aware of the need to take detailed histories from persons presenting with vague viral symptoms, especially those who have recently traveled, because missing a case of rabies will prove fatal for the affected patient.

Case Presentations

n Case OneA 22-year-old man presents to

the emergency department after being bitten by a dog while he was out jogging. He states that the dog came out of a yard and grabbed his right lower leg and when he pulled away the dog jumped up and bit his right thigh. The dog was a mixed breed with a collar on, and it ran away afterwards toward the same yard. He did not pursue the dog but knows the address where the attack occurred. His past medical history is positive for asthma, for which he uses an inhaler as needed. He has no known allergies, and his last tetanus update was 3 years ago. He has two 4-cm abrasions to his right posterior calf and a 1-cm puncture wound to his right lateral thigh that is bleeding slightly. Distal motor, sensory, and vascular functions are intact. The wounds are washed with soap and water and irrigated with povidone-iodine solution.

n Case TwoA 12-month-old girl is brought

to the emergency department by the girl’s mother because she found a bat in her daughter’s room, and

October 2009 • Volume 24 • Number 2

Page 14: Volume 24 • Number 2 In This Issue

Critical Decisions in Emergency Medicine

14

the pediatrician thought the child should be evaluated. The child has been healthy with normal milestones; she is on no medications and has had no prior hospitalizations. She has been acting normally, has had no change in her mental status, and has had no fever, rash, or illness. Her immunizations are current. She does have a few bruises from falling while playing. The bat was found on the floor under the girl’s bed. The mother brought the dead bat with her in a plastic container. The child’s examination shows a healthy, alert, playful child with a few typical knee abrasions/contusions and some superficial scratches on her face and neck. The child is able to walk, and her strength and balance are normal for her age.

n Case ThreeA 42-year-old game warden for the

state was bitten by a stray dog that he had been securing in a rural region of the state. The dog was quarantined and later euthanized because of erratic behavior; its remains were sent to the state pathologist for analysis. The patient has had rabies preexposure prophylaxis, but it has been more than 10 years, and his last antibody test was 5 years ago. He has a bite to his right lower leg over his shin that is 2 cm by 4 cm and a 1-cm puncture wound to the medial aspect of his calf. His past medical history is significant for previous animal bites and a forearm fracture 5 years ago. His motor, sensory, and vascular examinations are normal distal to the wound.

CRITICAL DECISIONWhich animals are known to carry the rabies virus?

Any carnivorous mammal can be a carrier of rabies. In the United States, most domestic animals are required to be vaccinated against the disease, so it is rare for a dog or cat to spread infection8,9; however, there is no way to measure compliance with these requirements. Cats typically have a higher incidence of rabies,

likely because there are more stray cats than stray dogs, and cats tend to be more nocturnal, which increases their risk for exposure to bats.3 In 2003, there were 321 reported cases of rabies in cats in the United States and only 117 reported cases in dogs.3 Since 1966, there have been only two documented cases of rabies transmissions from dogs to humans in this country.3 For those traveling in foreign countries, dogs are still the main source of infection.1 In this country, bats are the largest reservoir for the disease, accounting for almost 90% of infections acquired within the borders.1,9,10 Other wild animals known to harbor the disease in the United States are foxes, raccoons, and skunks. Small rodents can be infected by the disease if they are bitten by a rabid animal, but most die immediately from the inflicted wounds or are eaten. Transmission from a rodent to a human has never been documented.1 Rabies can also be passed from human to human in rare instances. Almost all of these cases involve transplanted organs.11,12 However, there are case reports of transmission through saliva involving human bites and extensive kissing.1

CRITICAL DECISIONWho should receive preexposure prophylaxis?

Anyone who has a high likelihood of being directly exposed to the virus should receive prophylaxis. Anyone working in a laboratory handling the rabies virus should be vaccinated before work is begun. The largest group affected in the United States is veterinarians, especially those working in remote areas with more likely exposure to wild animals. Those who work in animal control, fish and game workers, and spelunkers should also receive the vaccine. For the average American, rabies preexposure prophylaxis is only recommended for those traveling to and living for more than 1 month in an area where rabies is known to be endemic.3

Preexposure prophylaxis involves vaccination with 1 mL of cell culture vaccine on days 0, 7, and 28.1,3 The frequency of boosters is based on the actual risk of potential exposure. Those working with the virus should have titers drawn every 6 months. Those working in areas where rabies is known to be endemic should have levels drawn every 2 years. Those veterinarians and animal workers who choose to get the vaccine do not need to have titers monitored on a regular basis even if there is a known risk of rabies in their area, because there is no significant risk for rabies transmission.3 If these individuals do have a potential exposure, their titers can be drawn to determine need for treatment with further doses of the vaccine. If there is a significantly high risk of exposure, postexposure prophylaxis should be initiated until titer levels are available.

There are no known contraindications to preexposure prophylaxis. There have been numerous safety studies involving both the human diploid cell vaccine (HDCV) and the purified chick embryo cell vaccine (PCECV). There have been no reported deaths following administration of either vaccine. Pain around the injection site is the most common side effect. Mild systemic reactions, including headache and gastrointestinal symptoms, were seen in 10% to 50% of patients receiving the HDCV and in 0 to 30% of those receiving the PCECV. With repeat doses of HDCV, less than 0.001% of patients experienced systemic hypersensitivity reactions.3

CRITICAL DECISIONSWho should receive postexposure prophylaxis?

The primary route of rabies transmission is from a direct bite. It could also be spread if saliva, cerebrospinal fluid, or brain matter comes in contact with broken skin, and it can pass through mucous membranes.1,13 There have been

Page 15: Volume 24 • Number 2 In This Issue

October 2009 • Volume 24 • Number 2

15

multiple cases where rabies has been transmitted through organ transplants.11,12 There are also a few case reports that indicate aerosol transmission; however, all these cases have involved a large concentration of the virus, either in caves that are infested with bats or in a laboratory setting.1,13

Postexposure prophylaxis should be offered to anyone with a suspected exposure to rabies, especially anyone who was potentially exposed to a bat. Approximately 1% of bats have the rabies virus, and it is difficult to determine whether a person has been bitten by a bat.1 Bats teeth are so small and sharp that bats can bite a person without being noticed and the bite might not leave any identifiable mark behind. Everyone who suspects they have been bitten by a bat, those who find a dead bat in their room, and those who have camped outside in areas where bats are known to have rabies should receive immediate prophylaxis. All spelunkers who enter caves where bats are present should receive prophylaxis if they were not vaccinated prior to the potential exposure.1,3

Patients who have been bitten by wild carnivorous animals should also begin immediate postexposure prophylaxis. If the animal was caught, it should be shipped to an approved laboratory for testing. If the animal is found to be free of rabies, the patient may stop the vaccination schedule.3,8 Diagnosing rabies is done under immunofluorescent examination of the cerebral tissue.

Patients bitten by healthy appearing domestic animals may delay rabies postexposure prophylaxis if the animal is quarantined. These animals should be observed for 10 days, and if they show no sign of infection during the observation period they may be released, and the patient does not need to be vaccinated.1,3 If the animal shows any signs of infection, the patient should start the vaccination schedule and continue until the animal has been

tested at an approved facility. Signs of infection in an animal include excessive salivation, aggression, paralysis, daytime activity in nocturnal animals, and impaired movement.

As stated before, rodents have not been shown to pass the rabies virus to humans. Humans who have been bitten by chipmunks, beavers, woodchucks, etc., need not receive postexposure prophylaxis, although it is frequently given to allay fears of rabies.

To avoid person-to-person transmission via organ transplantation, no organs should be obtained from a person who died of an unexplained neurologic disease.1,3,11,12 If organs are procured from such a patient, immediate testing for the rabies virus should be performed. If anyone who has received an organ transplant presents with neurologic sequelae, rabies should be tested for, other potentially affected recipients should be notified, and the organ recipient should immediately be started on postexposure prophylaxis until the diagnosis is disproved.

There are no known contraindications for postexposure prophylaxis. Rabies is uniformly fatal; anyone with a potential exposure should begin immediate prophylactic treatment. As with the preexposure prophylaxis, patients can experience localized pain at the injection site, and a few will also experience mild systemic symptoms. There have been no serious side effects reported from administration of human rabies immunoglobulin (HRIG).1,3

CRITICAL DECISIONWhat is the schedule for rabies immunoglobulin and vaccine following a potential exposure?

The primary mode for preventing infection remains local wound care.1 Anyone presenting with an obvious bite should have it thoroughly irrigated. Although povidone-iodine cleansing solutions have the potential

to further damage tissue, they have been shown to be effective in killing the virus and should be used for bite wounds.3 As with any bite wound, the physician should allow it to heal by secondary intention, and sutures should not be used. The physician should consider approximating the wound if there is significant bleeding or significant exposure of underlying tissue or if the wound is located in a cosmetically significant area such as the face or hand.14

The most important feature of postexposure prophylaxis remains the immunoglobulin. This provides immunity to the virus while the body mounts its own response, triggered by the vaccine. Those who received preexposure prophylaxis need not receive the immunoglobulin, despite titer levels.2,3 Within the United States, the immunoglobulin is made from the serum of previously vaccinated humans. The dose is 20 units/kg.1-3,6,10 The immunoglobulin should be injected directly into the wound site. If the amount is too large to be injected into the affected area, the remainder should be given intramuscularly, usually in the deltoid muscle. For persons affected by multiple bites, the immunoglobulin may be diluted so that each bite site can be injected. Because of the expense and difficulty that is involved in producing HRIG, many other countries manufacture an equine version. This version is as effective as the human version in most studies but has a higher incidence of allergic reactions. The newer, purified form of equine vaccine has an almost undetectable rate of serum sickness, but the less purified version used in many countries because of cost issues is associated with a 15% to 40% incidence of serum sickness. The dose of equine immunoglobulin is 40 units/kg, and it should be administered in the same manner as the human version.3,6 Because of the current shortage of immunoglobulin, some hospitals in other countries do not offer it at all, which is

Page 16: Volume 24 • Number 2 In This Issue

Critical Decisions in Emergency Medicine

16

why preexposure prophylaxis is recommended for those traveling to areas where rabies is known to be endemic. The immunoglobulin has been shown to be effective up to 7 days after exposure, so travelers have time to get the appropriate treatment, even if it is not readily available in the community where they were bitten.

There are many forms of rabies vaccine available worldwide. The approved versions within the United States are the HDCV and the PCECV. Other countries have a wider variety of vaccines, derived from duck embryo, vero cells, and nerve tissues from affected animals. These vaccines may not be as effective, and patients returning to this country can require further treatment if they have not mounted a significant titer. The dose for all vaccines should be 1 mL,

given intramuscularly, preferably in the deltoid muscle, on days 0, 3, 7, 14, and 28.1-3,6,10 For those who had already received preexposure prophylaxis, vaccine need only be given as 1 mL, intramuscularly, on days 0 and 3, despite titer levels.1,3 Immunoglobulin has been shown to neutralize the vaccine, so different needles should be used for the administration of immunoglobulin and vaccine, and the vaccine should be given at a remote site to avoid interaction.3 Because rabies has a long incubation period, the vaccine schedule should be started even if an exposure is identified days to weeks after the fact.1,3,6 The schedule should be followed as if the day of presentation is day 0. If a patient misses a dose of the vaccine, the next dose should be given as soon as possible, and the schedule should be followed with appropriate time intervals from the new date.

CRITICAL DECISIONSHow is rabies definitively diagnosed?

Most cases of rabies are diagnosed in sacrificed animals. Any wild animal that was involved in a bite and any domestic animal that displays inappropriate behavior, including aggression, excessive salivation, paralysis, or impaired movement, while in quarantine should be killed and shipped to an approved laboratory for testing. These laboratories use an immunofluorescence technique to identify rabies antigen on cerebral tissues.15 This technique requires intact brain tissue, so care must be taken to preserve tissues while shipping the animal. To confirm a negative diagnosis, a tissue culture using mouse neuroblastoma cells is grown, and can be tested within a day.1,15 Many foreign countries do not have the laboratory facilities to undertake expensive diagnostic studies; however, the affected animal’s brain might be examined for typical cytoplasmic inclusion bodies within

the tissue. These inclusions have been found to have very low sensitivity, and there have been documented cases of human rabies after an animal has been deemed not to have rabies based on the microscopic examination of brain tissue.1

Diagnosing rabies in a living human subject is difficult, especially in the first few days following exposure. The most reliable test available at this time is PCR testing for rabies RNA in an affected person’s saliva.1,2,15 Skin biopsies are also routinely performed when trying to verify the diagnosis of rabies. The sample is usually taken from the neck, as a high concentration of nerve tissue around the hair follicles increases the chance of detection. These samples are then examined using the immunofluorescence technique to detect rabies antigen. The presence of rabies antibody in the serum of an individual who has received no previous vaccine or immunoglobulin is also diagnostic.1,15 Because of the low sensitivity of these tests, a person suspected of having rabies should be tested several times before the diagnosis is excluded.

CRITICAL DECISIONHow do patients with rabies infection present?

Rabies has a prolonged incubation period, from a few weeks up to many years.1,2,5,6,10,13 The incubation period varies depending on the severity of the exposure and the proximity of the bite to the central nervous system. Rabies spreads via retrograde fast axonal transport until it reaches the central nervous system, at which point it rapidly disseminates. Most affected patients will have some symptoms present within the first 3 months following exposure.

Early symptoms of rabies are usually associated with local reactions around the entry site. Patients might experience pain, itching, burning, or numbness in the affected area. Patients then develop constitutional symptoms, including malaise, fever,

PearlsAll patients potentially exposed •to a bat should be treated with rabies prophylaxis.

Patients who have received •preexposure prophylaxis do not need immunoglobulin after exposure.

Thorough wound care is •the first step in preventing rabies infection.

Waiting for official laboratory •testing should never delay prophylactic treatment following a potential exposure to rabies.

PitfallsInfiltrating immunoglobulin •too close to the vaccine can deactivate the vaccine.

Failure to quarantine a domestic •animal could result in prolonged unnecessary prophylaxis.

Patients presenting with •clinical signs of rabies have almost no chance of survival.

Page 17: Volume 24 • Number 2 In This Issue

October 2009 • Volume 24 • Number 2

17

cough, and anorexia. Because of the vague nature of the symptoms, most patients will be dismissed by health care workers as having a simple viral illness if a detailed history is not obtained.1,4,6,10

There are two defined subsets of clinical rabies presentations, the furious or encephalitic form and the paralytic form. The furious form is much more common. Patients affected by the furious form present with periods of hyperactivity, agitation, and hallucinations, mixed with periods of lucidity similar to delirium. They could experience spasms, most notably in response to stimuli. One well-defined symptom of rabies is hydrophobia—fear of water. The proposed reason for this is that patients experience spasms of the larynx when they attempt to drink water, and the body mounts a conditioned response to avoid the inciting trigger. Some patients have been documented to have such strong cases of hydrophobia that they experience spasms if they only think of water. The paralytic form of rabies is clinically indistinguishable from Guillain-Barré syndrome in many cases. Patients present with progressive paralysis that is irreversible. Both types of rabies progress to coma and autonomic instability, ultimately resulting in death.1,2,10,13 The mechanism of disease has yet to be described. Multiple studies have shown that rabies does not cause neuronal death, so neuronal dysfunction is the current theory behind the clinical manifestation of rabies.5

CRITICAL DECISIONWhat is the treatment for overt rabies infection?

Rabies is fatal once a person has developed clinical symptoms.1,2,4,6,10,13 There is no available treatment, and only one person who received no vaccine or immunoglobulin has ever been known to survive. The survivor was a 15-year-old girl who was treated with therapeutic coma and

antivirals. This case was documented in 2004, and the girl remains alive, although she suffers from moderate neurologic sequelae.16 This regimen has been repeated as an experimental treatment, but no further patients have survived. Once a person has symptoms, and a confirmed diagnosis is made, there is no proven benefit to life-saving measures.

Case Resolutions

n Case OneIn this case, the patient notified

local animal control officials, who quarantined the animal. The dog’s owner was able to show proof that the animal was currently vaccinated against rabies. At the end of the 10-day quarantine period, the dog had exhibited no abnormal behavior and was allowed to return home. The patient’s wound healed with only minimal scarring. Antibiotics were not judged necessary for this patient, since they are not indicated for leg wounds in immunocompetent patients. He had no long-term sequelae from the incident.

n Case TwoThe child in question received

HRIG and the first two doses of HDCV. The bat’s body was sent to the local health department laboratory for testing and was found to be free of rabies. The vaccination schedule was stopped, and the child had no long-term sequelae following the incident.

n Case ThreeA radiograph revealed no sign

of fracture or foreign body in the wound received by the game warden. The wound was cleansed with soap and water and irrigated with dilute povidone-iodine solution. Because the wound was gaping over the patient’s shin, it was reapproximated with a single layer of staples. This patient had received previous immunization against the rabies virus, so HRIG was not indicated. However, because the animal exhibited erratic behavior, the patient was given the recommended two doses of booster vaccine. His

wound healed without incident, and he experienced no long-term sequelae from the incident. It was recommended that he have titers drawn every 2 years because of the high-risk nature of his job.

SummaryEmergency physicians should

be aware of the incidence of rabies in their practice area. All animal bites should be reported to the local board of health. Most dogs in the United States have been vaccinated against rabies, and dogs with collars usually belong to someone who can be identified and notified to quarantine the animal. Cats are more at risk for rabies because of the lack of vaccination and because they are more often allowed to roam at night when they might come in contact with a rabid animal. An attempt should be made to identify and isolate an animal that has bitten someone. Rabies prophylaxis is not indicated if the animal can be located and remains healthy during the 10-day observation period.

Bats are the number one source of rabies in the United States, accounting for 90% of the reported cases. A bite from a bat can go unrecognized, especially in a small child. Postexposure prophylaxis should be initiated, and the bat should be sent to the local health department laboratory to be examined for rabies.

Although overt rabies is not a common presentation to emergency departments, many people will present seeking advice and treatment following an animal bite or exposure. Emergency physicians must be aware of the risks for rabies and take care to treat wounds appropriately and dispense vaccine and immunoglobulin in high-risk cases. Failure to do so may lead to the development of rabies, which is uniformly fatal.

References1. Wilkerson JA. Rabies. In: Auerbach B, ed. Wilderness

Medicine. 5th ed. Philadelphia, PA: Mosby/Elsevier; 2007:1206-1225.

Page 18: Volume 24 • Number 2 In This Issue

Critical Decisions in Emergency Medicine

18

2. Wyatt J. Rabies—update on a global disease. Pediatr Infect Dis J. 2007;26:351-352.

3. Manning SE, Rupprecht CD, Fishbein D, et al. Human rabies prevention—United States, 2008: recommendations of the Advisory Committee on Immunization Practices. MMWR Recomm Rep. 2008;57(RR-3):1-28.

4. Warrell MJ. Emerging aspects of rabies infection: with a special emphasis on children. Current Opin Infect Dis. 2008;21:251-257.

5. Fu ZF, Jackson AC. Neuronal dysfunction and death in rabies virus infection. J Neurovirol. 2005;11:101-106.

6. Takayama N. Rabies: a preventable but incurable disease. J Infect Chemother. 2008;14:8-14.

7. Centers for Disease Control and Prevention. Temporary unavailability of rabies pre-exposure vaccination. Online article updated May 20, 2008. Available at: http://www.cdc.gov/rabies/news/2008-05-20_PreEVax.html. Accessed March 31, 2009.

8. Lackay SN, Kuantg Y, Fu ZF. Rabies in small animals. Vet Clin Small Anim. 2008;38:851-861.

9. Belotto A, Leanes LF, Schneider MC, et al. Overview of rabies in the Americas. Virus Res. 2005;111:5-12.

10. Hankins DG, Rosekrans JA. Overview, prevention, and treatment of rabies. Mayo Clin Proc. 2004;79:671-676.

11. Bronnert J, Wilde H, Tepsumethanon V, et al. Organ transplantation and rabies transmission. J Travel Med. 2007;14:177-180.

12. Jackson AC. Rabies. Neurol Clin. 2008;26:717-726.

13. Jackson AC. Rabies: new insights into pathogenesis and treatment. Curr Opin Neurol. 2006;19:267-270.

14. Maimaris C, Quinton DN. Dog-bite lacerations: a controlled trial of primary wound closure. Arch Emerg Med. 1988;5:156-161.

15. Woldehiwet Z. Clinical laboratory advances in the detection of rabies virus. Clin Chim Acta. 2005;351;49-63.

16. Centers for Disease Control and Prevention. Recovery of a patient from clinical rabies—Wisconsin 2004. MMWR Morb Mortal Wkly Rep. 2004;53:1173-1175.

Page 19: Volume 24 • Number 2 In This Issue

October 2009 • Volume 24 • Number 2

19

The Drug BoxPralidoxime

By Michael Glueckert, MD; Summa Health System Emergency Medicine Residency

Organophosphate poisoning from pesticides is an uncommon problem in emergency medicine, but these potent cholinesterase inhibitors can cause severe cholinergic toxicity following cutaneous exposure, inhalation, or ingestion . They bind to acetylcholinesterase and render this enzyme nonfunctional . This inhibition leads to an overabundance of acetylcholine in the synapse . The dominant clinical features of acute cholinergic toxicity are bradycardia, miosis, lacrimation, salivation, bronchorrhea, bronchospasm, urination, emesis, and diarrhea . Treatment with pralidoxime and atropine can reverse these symptoms .

Pralidoxime

Mechanism of Action Reactivates cholinesterase inactivated by phosphorylation from exposure to organophosphate pesticides by displacing the enzyme from its receptor sites; removes the phosphoryl group from the active site of the inactivated enzyme .

Indications Organophosphate poisoning; acetylcholinesterase inhibitor toxicity; nerve agent toxicity management (unlabeled use)

Dosing Organophosphate poisoning – Initial: 30 mg/kg IV over 20 minutes; maintenance: 4-8 mg/kg/hour IV

Acetylcholinesterase inhibitor toxicity – Initial: 1-2 g IV followed by increments of 250 mg IV every 5 minutes until response observed

(may be given IM or SQ if IV is not feasible)

Side Effects Tachycardia, hypertension, dizziness, headache, drowsiness, rash, nausea, muscle rigidity, weakness, diplopia, hyperventilation, laryngospasm

Estimated Cost $400 (based on Summa Health System charges to patient)

Contraindication/Precautions Hypersensitivity to pralidoxime; poisonings due to phosphorus, phosphates, or pesticides of carbamate class (can increase toxicity of carbaryl)

Use with caution in patients with myasthenia gravis and in those with renal impairment

Should not be administered without concurrent atropine, to prevent worsening symptoms due to transient oxime-induced acetylcholinesterase inhibition

Page 20: Volume 24 • Number 2 In This Issue

Critical Decisions in Emergency Medicine

20

Page 21: Volume 24 • Number 2 In This Issue

October 2009 • Volume 24 • Number 2

21

The Critical Image

This case illustrates several important points:

Dog bites are forceful injuries capable of penetrating the skull, particularly in young children . Scalp wounds should be •evaluated for intracranial injury .

CT should be inspected on soft tissue windows to assess for intracranial hemorrhage and on bone windows to assess •for fracture .

Intracranial air (pneumocephalus) is more visible on bone windows, where it alone appears black . On this window •setting, all other tissues appear gray or white . On soft tissue windows, air appears nearly black but can be difficult to distinguish from other low-density substances including fat .

The patient was taken to the operating room for repair of injuries and recovered uneventfully .

Feature Editor: Joshua S . Broder, MD, FACEP

A 3-year-old girl presenting after an unprovoked attack by a pit bull. On arrival, the patient was awake and alert with no neurologic deficits, but she had multiple puncture wounds to the scalp. Noncontrast CT was obtained to assess for intracranial injury.

On soft tissue windows, fat (in this case,

intraorbital) looks nearly as black as air

Pneumocephalus (black)

Normal air in mastoid air cells

Normal air in ethmoid air cells

Fat appears gray on bone windows

Fracture

FractureNormal air in

mastoid air cellsPneumocephalus

(black)

Pneumocephalus (black)

}

Page 22: Volume 24 • Number 2 In This Issue

Critical Decisions in Emergency Medicine

22

Qualified, paid subscribers to Critical Decisions in Emergency Medicine may receive CME certificates for up to 5 ACEP Category I credits, 5 AMA PRA Category I Credits™, and 5 AOA Category 2-B credits for answering the following questions . To receive your certificate, go to www .acep .org/criticaldecisionstesting and submit your answers online . You will immediately receive your score and printable CME certificate . You may submit the answers to these questions at any time within 3 years of the publication date . You will be given appropriate credit for all tests you complete and submit within this time . Answers to this month’s questions will be published in next month’s issue .

CME Questions

1 . A 46-year-old man presents with shortness of breath . He is a maintenance worker at a local factory, and earlier in the day he had tightened a pipe fitting that was leaking “some sort of gas .” When asked, he states that the odor of the gas reminded him of a farm . His chest radiograph looks like that of someone with early adult respiratory distress syndrome, and he has mild, scattered wheezes . He requires 100% oxygen via nonrebreathing mask to maintain adequate oxygen saturation . Which of the following management strategies is most appropriate?A. albuterol nebulizer treatments 3, every 20 minutes, 125

mg methylprednisolone, and discharge with a prescription for prednisone, 20 mg PO three times daily for 4 days

B. albuterol nebulizer treatments 3 every 20 minutes, and admit to ICU

C. furosemide, 60 mg IV, and admit to ICUD. intravenous antibiotics, and admit to a telemetry bed on the

medical floorE. repeat chest radiograph in 4 hours, if normal discharge from

emergency department

2 . A 15-year-old boy was exposed to pepper spray about 2 hours ago . The nurse in the examination room is rubbing her red eyes and coughing . The patient is complaining of a burning sensation on his face and neck and is unable to open his eyes . His lung sounds are clear, and he has no complaints of respiratory difficulty . Which of the following is an appropriate next step?A. give him 50 mg of diphenhydramine intramuscularlyB. initiate decontaminationC. neutralize the irritant by applying baking soda to the patient’s

faceD. no treatment is necessaryE. use a bag valve mask and crush an amyl nitrate pearl near the

intake valve

3 . Which of the following is true regarding exposure to sulphur mustard gas?A. the earliest manifestations are often ocularB. early administration of a commercially available antidote kit will

prevent further clinical deteriorationC. exposure is identified by the characteristic early onset of

bullous blisteringD. thick canvas clothing will prevent vapor penetrationE. treatment with atropine eliminates all symptoms

4 . What effect should be anticipated from administration of botulinum antitoxin in a patient who has ingested botulinum toxin and is experiencing significant weakness of his respiratory muscles?A. the antitoxin should prevent further progression of the diseaseB. the antitoxin will prevent the patient from becoming

contagious to othersC. the antitoxin will reverse the paralysis that the patient has

developedD. no allergies have been reportedE. once patients progress to the point of requiring respiratory

support, the antitoxin is no longer indicated

5 . Which of the following is the most appropriate and effective treatment for a continual seizure that began 15 minutes after a nerve agent exposure?A. alprazolam orallyB. atropineC. midazolam intravenouslyD. physostigmineE. pyridoxine

6 . Which of the following is an accurate statement regarding cyanide exposure?A. flaccid paralysis is often the initial findingB. it is a cellular asphyxiantC. miosis is a common presentationD. pulmonary hypertension is a common cause of deathE. secretions should be treated with atropine

7 . Which of the following is the correct method for administering amyl nitrite pearls in a patient without intravenous access?A. administer the amyl nitrate for 30 minutes out of every hourB. crush the pearl and place it over the intake valve located on the

side of the bag valve maskC. an intact tablet should be placed over the intake valve located

on the side of the bag valve maskD. mix the amyl nitrite pearl with water and nebulize the solutionE. vascular access is essential, as amyl nitrate must be given

intravenously

8 . Which of the following is accurate regarding pepper spray?A. an alkylation reaction occurs binding to DNA and RNAB. binding to the acetylcholinesterase receptor elicits the effectsC. commonly created by the heating of Freon gasD. hydroxocobalamin is the appropriate treatment for acute

exposureE. the neurotransmitter substance P is released from peripheral

afferent sensory fibers

Page 23: Volume 24 • Number 2 In This Issue

October 2009 • Volume 24 • Number 2

23

9 . A patient presents with a bulbar paralysis and loss of head control . Which of the following exposures is most likely the cause?A. anthraxB. botulinum toxinC. cyanideD. phosgeneE. ricin

10 . Which of the following is the most common initial presentation of ricin intoxication?A. bronchorrheaB. cranial nerve palsyC. hypoxiaD. lacrimationE. nausea and colicky abdominal pain

11 . Which of the following describes the postexposure prophylaxis dosing schedule for human diploid cell vaccine (HDCV) for rabies exposure?A. 1 mL every other dayB. 1 mL on days 0, 3, 7, 14, 28C. 20 units/kg IV on day 0D. 20 units/kg on days 0, 3, 7, 14, 28E. depends on the weight of the patient

12 . A 19-year-old man is bitten by a rat while cleaning out his garage in Ohio . What is his risk for rabies?A. all wild animals are at high risk for rabiesB. rats are high-risk vectors of rabies; the patient should obtain

rabies immunizationC. rats are unlikely to carry rabies, and immunization is not

indicatedD. rats that are found in the Eastern United States are likely

carriersE. rodents in general usually do not carry rabies, but rats are the

exception

13 . Which of the following statements is true?A. the HDCV should be infiltrated as near as possible to the

wound siteB. the HDCV should only be given subcutaneouslyC. the site for HDCV infiltration should be different from the site

of human rabies immunoglobulin (HRIG) administrationD. the site for HDCV infiltration should be in the same extremity

as HRIGE. the site of rabies immunization is not important

14 . Which of the following is correct regarding clinical rabies?A. patients present with variable symptoms and signsB. patients present with vomiting and diarrhea primarilyC. symptoms develop within 7-10 days of inoculationD. symptoms develop within 3-4 weeks of inoculationE. symptoms develop within 30 days of inoculation

15 . Which of the following animals is most likely to carry rabies in the United States?A. catB. dogC. rabbitD. skunkE. squirrel

16 . Which of the following statements concerning rabies transmission is true?A. rabies can be transmitted via respiratory dropletsB. rabies can be transmitted via transplanted organsC. rabies is commonly transmitted from human to humanD. rabies is never found in catsE. the rabies source is usually identified in fatal cases

17 . The definitive diagnosis of rabies can be made by which of the following methods?A. observing a domestic animal for hydrophobiaB. observing a wild animal for 14 days for signs of rabiesC. performing a brain biopsy of any suspected patient and

looking for Negri bodiesD. performing an immunofluorescent antibody test on the

patient’s urineE. performing PCR testing for rabies RNA on the patient’s saliva

18 . Clinical rabies can present as:A. the encephalitic form in humansB. the encephalitic/furious form or the paralytic formC. the paralytic form in adultsD. the paralytic form in childrenE. viral gastroenteritis

19 . The case fatality rate for clinical rabies is:A. 10%-20%B. 20%-50%C. 50%-75%D. 99%-100%E. dependent on comorbid factors

20 . Which of the following is correct regarding rabies?A. it is common in dogs in the United StatesB. it is a DNA virusC. it is rarely vaccinated for in the United StatesD. it is transmitted via bites by wild animals primarilyE. it is transmitted via needlestick

Answer key for September 2009, Volume 24, Number 1

The American College of Emergency Physicians makes every effort to ensure that contributors to College-sponsored publications are knowledgeable authorities in their fields. Readers are nevertheless advised that the statements and opinions expressed in this series are provided as guidelines and should not be construed as College policy unless specifically cited as such. The College disclaims any liability or responsibility for the consequences of any actions taken in reliance on those statements or opinions. The materials contained herein are not intended to establish policy, procedure, or a standard of care.

1 2 3 4 5 6 7 8 9 10 11 12 13 14 15 16 17 18 19 20D A E D D D B B A A E C C D E C C C A A

Page 24: Volume 24 • Number 2 In This Issue

NONPROFIT

U.S. POSTAGE

P A I D

DALLAS, TX

PERMIT NO . 1586

The Critical ECG

October 2009 • Volume 24 • Number 2Critical Decisions in Emergency Medicine is the official CME publication of the American College of Emergency Physicians. Additional volumes are available to keep emergency medicine professionals up-to-date on relevant clinical issues.

Editor-in-ChiefLouis G. Graff IV, MD, FACEP

Professor of Traumatology and Emergency Medicine, Professor of Clinical Medicine, University of

Connecticut School of Medicine; Farmington, Connecticut

Section EditorJ. Stephen Bohan, MS, MD, FACEP

Executive Vice Chairman and Clinical Director, Department of Emergency Medicine, Brigham &

Women’s Hospital; Instructor, Harvard Medical School,Boston, Massachusetts

Feature EditorsMichael S. Beeson, MD, MBA, FACEP

Program Director, Department of Emergency Medi-cine, Summa Health System, Akron, Ohio; Professor,

Clinical Emergency Medicine, Northeastern Ohio Universities College of Medicine, Rootstown, Ohio

Joshua S. Broder, MD, FACEP Assistant Clinical Professor of Surgery, Associate

Residency Program Director, Division of Emergency Medicine, Duke University Medical Center, Durham,

North Carolina

Amal Mattu, MD, FACEPProgram Director, Emergency Medicine

Residency Training Program,Co-Director, Emergency Medicine/Internal Medicine Combined Residency Training Program, University of Maryland School of Medicine, Baltimore, Maryland

Associate EditorsDaniel A. Handel, MD, MPH

Director of Clinical Operations, Department of Emergency Medicine, Oregon Health & Science University, Portland,

Oregon

Frank LoVecchio, DO, MPH, FACEPResearch Director, Maricopa Medical Center Emergency

Medicine Program; Medical Director, Banner Poison Control Center, Phoenix, Arizona; Professor, Midwestern

University/Arizona College of Osteopathic Medicine, Glendale, Arizona.

Sharon E. Mace, MD, FACEPAssociate Professor, Department of Emergency

Medicine, Ohio State University School of Medicine; Faculty, MetroHealth Medical Center/Cleveland Clinic Foundation Emergency Medicine Residency Program; Director, Pediatric Education/Quality Improvement and Observation Unit, Cleveland Clinic Foundation,

Cleveland, Ohio

Robert A. Rosen, MD, FACEPMedical Director, Culpeper Regional Hospital,

Culpeper, Virginia

George Sternbach, MD, FACEPClinical Professor of Surgery

(Emergency Medicine), Stanford University Medical Center, Stanford, California

Editorial StaffMary Anne Mitchell, ELS

Managing Editor

Mike GoodwinCreative Services Manager

Jessica HamiltonEditorial Assistant

Lilly E. FriendCME and Subscriptions Coordinator

Marta FosterDirector and Senior Editor

Educational and Professional Publications

Critical Decisions in Emergency Medicine is a trademark owned and published monthly by the American College of Emergency Physicians, PO Box 619911, Dallas TX 75261-9911. Send address changes to Critical Decisions in Emergency Medicine, PO Box 619911, Dallas TX 75261-9911, or to [email protected].

Copyright 2009 © by the American College of Emergency Physicians. All rights reserved. No part of this publication may be reproduced, stored, or transmitted in any form or by any means, electronic or mechanical, including storage and retrieval systems, without permission in writing from the Publisher. Printed in the USA.

[email protected]

Sinus rhythm with sinus arrhythmia, rate 75, prolonged QT. The major abnormality found in this ECG is the markedly prolonged QT interval (QT interval 0 .520 seconds, QTc interval 0 .581 seconds) . Prolonged QT interval is associated with hypokalemia, hypomagnesemia, hypocalcemia, acute myocardial ischemia, elevated intracranial pressure, drugs with sodium channel blocking effects (for example the cyclic antidepressants, quinidine, etc .), hypothermia, and congenital prolonged QT syndrome . In this case, the abnormality was caused by an overdose of one of the patient’s antipsychotic medications . These medications, as well as many others, are associated with the prolonged QT interval and the potential for developing torsade de pointes .

Feature Editor: Amal Mattu, MD, FACEP

From: Mattu A, Brady W . ECGs for the Emergency Physician. London: BMJ Publishing; 2003:102,141 . Available at www .acep .org/bookstore . Reprinted with permission .

A 57-year-old man with a history of schizophrenia presenting after an overdose.